返回列表 发帖

old prep cr2-58

Theproposal to hire ten new police officers in Middletown is quite foolish.  There is sufficient funding to pay thesalaries of the new officers, but not the salaries of additional court andprison employees to process the increased caseload of arrests and convictionsthat new officers usually generate.

Whichof the following, if true, will most seriously weaken the conclusion drawnabove?

(A)Studies have shown that an increase in a city's police force does notnecessarily reduce crime.
(B)When one major city increased its police force by 19 percent last year, therewere 40 percent more arrests and 13 percent more convictions.
(C)If funding for the new police officers' salaries is approved, support for othercity services will have to be reduced during the next fiscal year.
(D)In most United States cities, not all arrests result in convictions, and notall convictions result in prison terms.(E)
(E)Middletown's ratio of police officers to citizens has reached a level at whichan increase in the number of officers will have a deterrent effect on crime.
我的问题是:

我理解的是文中的 conclusion是:Theproposal to hire ten new police officers in Middletown is quite foolish,

原因是:There is sufficient funding to pay thesalaries of the new officers, but not the salaries of additional court andprison employees to process the increased caseload of arrests and convictionsthat new officers usually generat。
照我这样的理解思路答案是D,可是文中答案是E,我却认为E是加强了结论。我知道我的思路肯定是错了,可是怎么也拐不到正确的答案上去。求指点!!!
收藏 分享

首先,楼主找的结论是没有问题的,文章的结论就是第一句,新增加警务人员是没有必要的。

首先来看D:原文对结论的论述过程是说有钱增加警力,却没有钱来完成后续的审判和关押等新问题。而D对此的反驳是说:并不是所有被抓捕的人都会被定罪,也不是所有被定罪的人都会被关押。这个确实可以说是一个削弱,但要注意的是,需要和其他选项比较,看它是不是最强的削弱。(因为即使按照哦这个选项的说法,被定罪和被关押的人数还是会增加,那是否有足够的资金支持这部分还是不得而知)。

再来看E选项,说现在警员的比例已经到了一个层次,在这个层次上再增加警力的话会对犯罪有威慑作用。这是一个很强的削弱,由于威慑作用可以有效的减少犯罪,这样的话不但不会增加后续的定罪和关押,还能减少这一部分的数量,就完全不用担心资金不足的问题了,所以增加警力数量并不是没有必要的,因而E选项是最佳选项。

TOP

返回列表

站长推荐 关闭


美国top10 MBA VIP申请服务

自2003年开始提供 MBA 申请服务以来,保持着90% 以上的成功率,其中Top10 MBA服务成功率更是高达95%


查看